LSAT and Law School Admissions Forum

Get expert LSAT preparation and law school admissions advice from PowerScore Test Preparation.

 Administrator
PowerScore Staff
  • PowerScore Staff
  • Posts: 8916
  • Joined: Feb 02, 2011
|
#40382
Complete Question Explanation
(The complete setup for this game can be found here: lsat/viewtopic.php?t=13080)

The correct answer choice is (E)

This question stem establishes that V is shown third, which is only possible in Templates 2.1 and 2.2. In both of them T must be shown fifth—validating answer choice (E).

Without the use of templates, we need to split the solution into two directions. One way to do this would be to take T and place it in either the first or the fifth positions:
  • If T is first, then R must be second. However, this would force Q and S to be adjacent to each other, in violation of the last rule:
PT72_Game_#2_#11_diagram 1.png
Clearly, since T cannot be first, it must be fifth, in accordance with the second rule of the game.
You do not have the required permissions to view the files attached to this post.

Get the most out of your LSAT Prep Plus subscription.

Analyze and track your performance with our Testing and Analytics Package.